Ile jest macierzy takich, że

Przestrzenie wektorowe, bazy, liniowa niezależność, macierze.... Formy kwadratowe, twierdzenia o klasyfikacji...
Bobi02
Użytkownik
Użytkownik
Posty: 213
Rejestracja: 6 paź 2013, o 22:39
Płeć: Mężczyzna
Lokalizacja: Warszawa
Podziękował: 22 razy
Pomógł: 3 razy

Ile jest macierzy takich, że

Post autor: Bobi02 »

Witam. Walczę z takim problemem. Próbowałem dojść do sedna w dość prymitywny sposób, czyli rozpatrując różne przypadki. Jednak to dość żmudne, zna ktoś subtelniejsze rozwiązanie?

Ile jest macierzy \(\displaystyle{ A \in M_{10x10}(\mathbb{R})}\) złożonych z liczb 0 i 1 takich, że liczba \(\displaystyle{ det A}\) jest nieparzysta?
Awatar użytkownika
rtuszyns
Użytkownik
Użytkownik
Posty: 2042
Rejestracja: 29 gru 2006, o 23:24
Płeć: Mężczyzna
Lokalizacja: Zamość
Podziękował: 1 raz
Pomógł: 229 razy

Ile jest macierzy takich, że

Post autor: rtuszyns »

Napisz warunek na nieparzystość wyznacznika. Wyznacznik obliczyć można stosując rozwinięcie Laplace'a.
Bobi02
Użytkownik
Użytkownik
Posty: 213
Rejestracja: 6 paź 2013, o 22:39
Płeć: Mężczyzna
Lokalizacja: Warszawa
Podziękował: 22 razy
Pomógł: 3 razy

Ile jest macierzy takich, że

Post autor: Bobi02 »

Najlepiej wyjść od macierzy jedynek, czy to nie ma znaczenia?
Awatar użytkownika
Zordon
Użytkownik
Użytkownik
Posty: 4977
Rejestracja: 12 lut 2008, o 21:42
Płeć: Mężczyzna
Lokalizacja: Kraków
Podziękował: 75 razy
Pomógł: 910 razy

Ile jest macierzy takich, że

Post autor: Zordon »

To jest równoważne pytaniu ile jest odwracalnych macierzy \(\displaystyle{ n \times n}\) nad ciałem \(\displaystyle{ \ZZ_2}\). Odpowiedź na to pytanie to \(\displaystyle{ (2^n-1)(2^n-2)\cdot ...\cdot (2^n-2^{n-1})}\).
A rozumowanie jest takie:
1. pierwszą kolumnę wybieramy dowolnie, żeby tylko nie była zerem \(\displaystyle{ (2^n-1)}\) opcji
2. drugą kolumnę musimy wybrać tak, żeby była liniowo niezależna wspólnie z pierwszą, można to zrobić na \(\displaystyle{ (2^n-2)}\) sposoby
3. i-tą kolumnę musimy wybrać tak, aby była lnz, z pierwszymi (i-1) kolumnami, zatem ma nie leżeć w przestrzeni \(\displaystyle{ V_{i-1}}\) rozpiętej przez pierwszych (i-1) kolumn. Mamy \(\displaystyle{ \dim(V_{i-1})=i-1}\), zatem \(\displaystyle{ |V_{i-1}|=2^{i-1}}\). Zatem mamy \(\displaystyle{ (2^n-2^{i-1})}\) możliwości.


edit: chociaż jestem ciekawy jak to robić rozwinięciem Laplace'a, tak jak sugeruje rtuszyns wyżej.
ODPOWIEDZ